¿Dónde se usa el axioma de elección en la demostración de Rudin de que "la unión contable de conjuntos contables es contable"?

Baby Rudin demuestra que la unión contable de conjuntos contables es contable. Al leer otras pruebas en línea, se debe invocar el axioma de elección; sin embargo, no veo inmediatamente dónde se usa eso aquí.

Teorema 2.12. Dejar { mi norte } , norte = 1 , 2 , . . . ser una secuencia de conjuntos contables, y poner S = norte = 1 mi norte . Entonces S es contable.

Prueba. Deja que cada conjunto mi norte estar ordenado en una secuencia { X norte k } , k = 1 , 2 , 3 , , y considere la matriz infinita

IMG

en el que los elementos de mi norte formar el norte tirar. La matriz contiene todos los elementos de S. Como indican las flechas, estos elementos se pueden organizar en una secuencia

( 17 )       X 11 ; X 21 , X 12 ; X 31 , X 22 , X 13 ; X 41 , X 32 ;
Si dos de los conjuntos mi norte tienen elementos en común, estos aparecerán más de una vez en (17). Por lo tanto, hay un subconjunto T del conjunto de todos los enteros positivos tales que S T , lo que demuestra que S es a lo sumo contable. Desde mi 1 S , y mi 1 es infinito, S es infinito, y por lo tanto contable.

¿"Recoger" los elementos de cada mi norte y ordenarlos en una secuencia invoca AC? Eso no me parece bien, desde luego parece el clásico" q es contable" también invocaría la elección. ¿Está AC oculto en el argumento "hay un subconjunto T del conjunto de todos los enteros positivos tales que S T , lo que demuestra que S es a lo sumo parte contable"?

Muchas gracias. Cualquier ayuda/percepción sería realmente apreciada.

"el clásico " q es contable" argumento también invocaría elección" - por lo general exhibe una enumeración particular de q para argumentar esto. No dices, "las fracciones con denominador 1 son contables, y también lo son aquellas con denominador 2, y así sucesivamente para cada denominador norte ". Pero si solo dijeras eso, estarías exactamente en el mismo barco, y necesitarías el Axioma de Elección (Contable).

Respuestas (2)

Sí, necesita elegir para elegir una secuencia particular para cada uno de los mi norte s -- recuerda que todo lo que sabes sobre ellos para el propósito de esta prueba es que existe al menos una secuencia; esto deja abierta la posibilidad de que haya muchas secuencias y ninguna forma de principio para seleccionar una en particular entre ellas.

Se supone que hay alguna forma de organizar cada mi norte en una fila, pero lo que necesitas para construir una biyección F : norte S es que usas la misma enumeración de mi 1 cuando defines F ( 1 ) como cuando defines F ( 3 ) , F ( 6 ) , F ( 10 ) , etcétera. Del mismo modo, es necesario utilizar la misma enumeración de mi norte cada vez que elija un número de él. Es recordar una elección particular de secuencia para cada uno de los infinitos mi norte s que requiere que coloque esas opciones en un solo objeto matemático, y luego necesita el Axioma de Elección para asegurarse de que dicho objeto existe.

No te encuentras con este problema al mostrar que q es contable, porque allí sabe lo suficiente sobre los conjuntos que puede especificar una secuencia particular para usar en cada caso y, por lo tanto, no hay necesidad de tomar decisiones arbitrarias.

Demostración q es contable no requiere AC. Demostrar que una unión contable de conjuntos contables es contable requiere alguna forma de AC.

Sí, AC se usa justo al principio, donde dice que cada elemento de la secuencia, etc.

Dicho mi 1 es contable dice que los elementos de mi 1 puede organizarse en una secuencia. Hay muchas maneras de hacer esto; tenemos que elegir uno. También tenemos que elegir uno para mi 2 y uno para mi 3 ... tenemos infinitas "opciones" que hacer.

Demostrando que norte es contable no requiere tales "elecciones", porque, digamos, los elementos de la forma ( 1 , metro ) ya están dispuestos en una secuencia para nosotros.

Pero si los elementos de E no pueden ordenarse en secuencia, ¿puede E ser contable?
@martycohen: Sí, por supuesto que se pueden organizar en secuencia. El problema es que hay muchas formas de hacerlo, y para obtener una secuencia de mi norte , debe asegurarse de utilizar la misma secuencia para mi 1 cuando defines F ( 1 ) como cuando definiste F ( 3 ) , F ( 6 ) , F ( 10 ) , y de manera similar para cada una de las otras filas. Es recordar una elección particular de secuencia para cada uno de los infinitos mi norte s que requiere que coloques esas opciones en un conjunto, y luego necesitas el Axioma de Elección para asegurarte de que ese conjunto existe.
@marty: recuerde que lo que dice el Axioma de Elección es (equivalente a) "el producto cartesiano de conjuntos no vacíos no es vacío". En este caso, tenemos el conjunto de enumeraciones de mi 1 (llámalo PAG 1 ), el conjunto de enumeraciones de mi 2 , etc., y cualquier elemento del producto de esos contablemente muchos PAG i indexado usando norte , conduce a una enumeración de mi y hemos terminado. Entonces, lo que necesitamos es el "hecho" "obvio" (por lo tanto, un axioma aceptable, al menos uno así lo espera...) de que el producto infinito de esos PAG i conjuntos no es vacío.